Search found 17 matches


Logitech on different note your signature is pretty interesting :)

by ab78

Wed Oct 29, 2008 5:35 am
Forum: Data Sufficiency
Topic: OG Quant Review question - I don't quite get this
Replies: 9
Views: 1881

Thanks everyone for the reply. I appreciate the responses and I now know where I went wrong.

by ab78

Wed Oct 29, 2008 5:33 am
Forum: Data Sufficiency
Topic: OG Quant Review question - I don't quite get this
Replies: 9
Views: 1881

OG Quant Review question - I don't quite get this

If xy > 0 , does (x-1)(y-1) = 1 ? 1. x+y = xy 2. x=y OA = A I got it as D and I can explain why. Don't know if I am doing some stupid mistake. 1 . Is sufficient pretty straightforward. 2. (y-1)(y-1)=1 thus y^2 - 2y=0 thus we get either y=0 or y=2. since they have mentioned xy>0 y can't be zero, it h...

by ab78

Tue Oct 28, 2008 3:50 am
Forum: Data Sufficiency
Topic: OG Quant Review question - I don't quite get this
Replies: 9
Views: 1881

yeah it is A :). No I don't have a link, my friend sent me this as a word doc.

by ab78

Sat Oct 25, 2008 5:18 pm
Forum: Critical Reasoning
Topic: LSAT CR
Replies: 11
Views: 4104

Yeah I had picked B too. But OA seems to be A. This question is from GMAT-LSAT CR Doc that I have. I think it is same as 1000CR.

Do you want to try second one I will post the OA later.

by ab78

Sat Oct 25, 2008 10:53 am
Forum: Critical Reasoning
Topic: LSAT CR
Replies: 11
Views: 4104

LSAT CR

It is illogical to infer a second and different effect from a cause which is known only by one particular effect. This is incorrect because the inferred effect must necessarily be produced by some different characteristic of the cause than is the observed effect, which already serves entirely to des...

by ab78

Sat Oct 25, 2008 5:19 am
Forum: Critical Reasoning
Topic: LSAT CR
Replies: 11
Views: 4104

It can't definitely be B because it changes the meaning of the argument you can't renew the past, it can be reinterpreted instead as mentioned in the argument.

I had doubt between A & C, but A makes much more sense.

by ab78

Wed Oct 22, 2008 12:00 pm
Forum: Critical Reasoning
Topic: History textbooks frequently
Replies: 12
Views: 2994

I was following the post because I had picked up D and everyone else was saying B and I don't believe myself with CR because I am extremely bad with it :(

you guys who are good with CR can you recommend me some good strategies?

by ab78

Tue Oct 21, 2008 5:47 pm
Forum: Critical Reasoning
Topic: The wild mouflon sheep
Replies: 13
Views: 7062
by ab78

Tue Oct 21, 2008 6:02 am
Forum: Problem Solving
Topic: routes and percentages
Replies: 8
Views: 1495

Doesn't "C" also sound good. The tense is past so we stick to simple tense which is prefered hence was, but should created by and erupted be in paralle with each other?

Am I missing something here?

by ab78

Mon Oct 20, 2008 8:00 am
Forum: Sentence Correction
Topic: mgmatsc1
Replies: 12
Views: 2037

Just to help with more clarification this is a conditional probability problem. P(Beer)=P(Beer intersection Mixed Drinks) /P(Mixed Drinks) essentially you can write it as P(Beer) = # of students drinking both /# of students drinking mixed drinks why?? because total # of students is the same in both ...

by ab78

Mon Oct 13, 2008 7:34 am
Forum: Problem Solving
Topic: Need clarification
Replies: 7
Views: 1694

aah here is where i went wrong : while picking consultants I picked up

5C4 + 5C3..should have done "and" with 2C1 in the second part.

that was a good explanation.

by ab78

Sun Oct 12, 2008 8:08 am
Forum: Problem Solving
Topic: Tricky Combination Problem
Replies: 6
Views: 1993
by ab78

Sat Oct 11, 2008 4:21 pm
Forum: Problem Solving
Topic: Tricky Combination Problem
Replies: 6
Views: 1993

P(not E or not F) if were mutually exclusive then it is true that P(not E)+ P(not F) = 0.25 1-P(E) + 1-P(F) = 0.25 2-0.25 = P(E) + P(F) (exclusive E & F) events. comes out to be greater than 1 . Probability can't be greater than 1 so it s not exclusive. There has to be some intersection between ...

by ab78

Sat Oct 11, 2008 2:37 pm
Forum: Problem Solving
Topic: probability
Replies: 6
Views: 1687

awesome..I had come across this problem in Kaplan I think. They have given a very crude solution to this. But I like your solution better. Helped me too. Thanks

by ab78

Sat Oct 11, 2008 2:21 pm
Forum: Problem Solving
Topic: Sets Problem
Replies: 5
Views: 1641